Has descubierto un famoso problema en termodinámica.
En nuestro caso el pistón no se moverá. El argumento mecánico es correcto, mientras que el argumento de la entropía máxima no es concluyente.
Para ver que $P_1=P_2$ es una posición de equilibrio también se puede aplicar la conservación de la energía. Como no hay intercambio de calor,
$$dU_{1,2} = -P_{1,2} dV_{1,2}$$
Requerimos que $dU=0$ ya que nuestro sistema está aislado del entorno, por lo que
$$dU_1 + dU_2 = 0 \to P_1 d V_1 + P_2 dV_2 = 0$$
Pero $V=V_1+V_2$ y $V$ es fijo, por lo que $dV_1 = - dV_2$ y obtenemos
$$P_1=P_2$$
Ahora veamos el principio del máximo de entropía. El problema es que has olvidado que $S$ también es una función de la energía:
$$S(U,V)= S_1 (U_1, V_1)+ S_2 (U_2, V_2)$$
$$d S = dS_1 + dS_2 = \frac{dU_1}{T_1} + \frac{P_1}{T_1} dV_1 + \frac{dU_2}{T_2} + \frac{P_2}{T_2} dV_2$$
Desde $dU_{1,2} = -P_{1,2} dV_{1,2}$ vemos que $dS$ desaparece idénticamente, por lo que no podemos decir nada sobre $P_{1,2}$ y $T_{1,2}$ El principio del máximo de entropía no es, pues, concluyente.
Actualización
Tu pregunta me ha inspirado muchas reflexiones en los últimos días y he descubierto que... Me equivoqué . Básicamente seguí el argumento dado por Callen en su libro Termodinámica (Apéndice C), pero parece:
- Hay algunos problemas con el propio argumento
- He interpretado mal el argumento
Mi error fue realmente tonto: Sólo mostré que $P_1=P_2$ es un necesario condición para el equilibrio, no es que sea una suficiente condición, es decir, (si el argumento es correcto y) si el sistema está en equilibrio, entonces $P_1=P_2$ pero si $P_1=P_2$ el sistema podría seguir estando en desequilibrio... ¡que lo está!
Todavía no soy capaz de explicar por qué todo el argumento es erróneo: algunos autores han dicho que las consideraciones de equilibrio deben seguirse de la segunda ley y no de la primera y que la segunda ley es no no es concluyente. Se puede leer, por ejemplo este artículo y este artículo . Ambos utilizan sólo consideraciones termodinámicas, pero te advierto que el segundo trata de contradecir al primero. Así que el problema, desde un punto de vista puramente termodinámico, es realmente difícil de resolver sin cometer errores, y no he encontrado ningún argumento que me convenza del todo y para siempre.
Este artículo tiene en cuenta exactamente su problema y muestra que el pistón se movimiento, haciendo la suposición adicional de que los gases son ideal gases.
Tomamos las temperaturas iniciales, T1 y T2, como diferentes, y las presiones iniciales, p1 y p2, como iguales. Una vez desbloqueado, el pistón gana una energía de traslación hacia la derecha del orden 1/2KT1 por una colisión con una molécula del lado 1, y una energía de traslación hacia la izquierda del orden 1/2KT2 por una colisión con una molécula del lado 2 . De este modo, la energía pasa principalmente del lado 2 al lado 1 si T2>T1.
[...] En este proceso que acabamos de considerar, las presiones en los dos lados del pistón son iguales en todo momento, lo que significa que no se realiza ningún "trabajo". Sin embargo, la transferencia de energía se produce a través de la agencia del pistón en movimiento, y si se considera que el "trabajo" es la energía transferida a través de macroscópico, no aleatorio, entonces parece que se realiza "trabajo".
Esto es realmente similar al argumento dado por Feynman en su conferencias (39-4). Feynman utiliza básicamente argumentos de la teoría cinética para demostrar que si empezamos con $P_1 \neq P_2$ el pistón al principio "chapoteará de un lado a otro" (cit.) hasta $P_1 = P_2$ y luego, debido al azar fluctuaciones de presión convergen lentamente hacia el equilibrio termodinámico ( $T_1=T_2$ ).
El argumento es realmente complicado porque suponemos que si la presión es la misma en ambos lados el pistón no se moverá, olvidando que la presión es sólo $2/3$ de la densidad por la energía cinética media por partícula
$$P = \frac 2 3 \rho \langle \epsilon_K \rangle$$
al igual que la temperatura es básicamente la energía cinética media (sin el factor multiplicador de la densidad). Así que estamos tratando con estadística cantidades, que no son "constantes" desde el punto de vista microscópico. Así, mientras que termodinámicamente decimos que si $P_1=P_2$ el pistón no se moverá, desde un punto de vista microscópico en realidad será ligeramente meneo hacia adelante y hacia atrás debido a la diferente colisión que experimenta de las partículas en los lados izquierdo y derecho.
También ha habido simulaciones de su problema que muestran que si empezamos con $P_1=P_2$ y $T_1\neq T_2$ el pistón oscilará hasta alcanzar el equilibrio termodinámico ( $T_1=T_2$ ). Vea las fotos de abajo, que tomé del artículo.
1 votos
Los dos tratamientos no son contradictorios. Si sólo se deja que los sistemas intercambien trabajo mecánico (es decir, que el pistón sea un aislante perfecto), nunca llegarán al equilibrio termodinámico.
2 votos
Pero la pregunta es en realidad bastante complicada. Se trata de indeterminado cuánto se moverá el pistón; la termodinámica por sí sola no puede decirlo.
1 votos
Si se considera el equilibrio de fuerzas en el pistón liberado, el pistón puede estar en equilibrio de fuerzas sólo si las dos presiones son iguales. La única manera de que eso ocurra es que el pistón no se mueva.
0 votos
La respuesta es la primera, no por la termodinámica sino por las leyes de newton. Sin embargo, no puedo seguir la lógica del segundo argumento.
2 votos
Mira el enlace publicado en la respuesta aceptada: physics.stackexchange.com/questions/105344/
0 votos
Una pregunta interesante relacionada con esto: ¿se puede en realidad unir los dos sistemas sin cambiar la entropía del sistema de $S_1+S_2$ . Ese momento en el que se juntan no es un estado de equilibrio, por lo que puede ser difícil definir la entropía "correctamente". La entropía se define más fácilmente en el equilibrio termodinámico.
0 votos
@tonydo: sí, esta pregunta parece un duplicado exacto de aquella. (Pero no se puede cerrar debido a la recompensa abierta).